PDA

Archiv verlassen und diese Seite im Standarddesign anzeigen : Quantum computing: Messung in anderer Basis


wry
2007-09-20, 21:09:12
Hallo erstmal,

ich hab momentan ein Problem bei einer Aufgabe und wer weiß, vielleicht ist hier im Forum ja jemand der sich mit Quantum Computing ein bisschen auskennt. :smile:

Es geht um eine Messung eines 2-qubit Systems das in Basis X gegeben ist und ich soll die Wahrscheinlichkeiten der Messergebnisse für Basis Y angeben.

Kennt sich jemand damit aus? Ein guter Link würde mir vielleicht auch schon helfen, find dazu nichts gescheites.

Ich kann auch gerne ein Beispiel posten, falls das jemand wünscht,
aber wollte erstmal schauen ob sich wer damit auskennt :redface:

Gast
2007-09-20, 23:11:43
Hallo erstmal,

ich hab momentan ein Problem bei einer Aufgabe und wer weiß, vielleicht ist hier im Forum ja jemand der sich mit Quantum Computing ein bisschen auskennt. :smile:

Es geht um eine Messung eines 2-qubit Systems das in Basis X gegeben ist und ich soll die Wahrscheinlichkeiten der Messergebnisse für Basis Y angeben.

Kennt sich jemand damit aus? Ein guter Link würde mir vielleicht auch schon helfen, find dazu nichts gescheites.

Ich kann auch gerne ein Beispiel posten, falls das jemand wünscht,
aber wollte erstmal schauen ob sich wer damit auskennt :redface:

Messen????????????????? War da nicht was mit unschärfe?

Nicht, daß ich dir Softwaretechnisch großartig weiterhelfen könnte (wir haben seinerzeit diese Mimik mit Pfeil hoch bzw. runter aufm Blatt Papier beschrieben), ein Beispiel und/oder weitere Erläuterungen würden mich aber dennoch interessieren.

Trap
2007-09-20, 23:45:14
Ja, das gehört eigentlich nicht ins Programmier-Forum.

Der eine Teil ist reine lineare Algebra, die Basistransformation. Der andere Teil ist, dass die Wahrscheinlichkeit eines Messwerts dem Skalarprodukt des zugehörigen Basisvektor mit dem Zustandsvektor entspricht.

Gast
2007-09-20, 23:48:23
nimm an, die Basiszustände in Basis X seien { |X_i> } (da du 2 QBits hast, gibt es 4 Basiszustände, daher kann i von 1 bis 4 laufen). Der Zustand |psi> des Systems in Basis X dargestellt ist dann

|psi> = sum_i=1^4 a_{X,i} |X_i>

Seien nun { |Y_j> } die Basiszustände in Basis Y, dann ist die Wahrscheinlichkeit, das System in einem Zustand |Y_j> vorzufinden:

P_{Y,j} = <Y_j|psi> = sum_i=1^4 a_{X,i} <Y_j|X_i>

Gast
2007-09-20, 23:50:00
Messen????????????????? War da nicht was mit unschärfe?die Unschärferelation besagt nur, daß der Meßwert nicht genau vorgesagt werden kann, nicht aber daß gar nicht gemessen werden könne.

Gast
2007-09-20, 23:58:01
die Unschärferelation besagt nur, daß der Meßwert nicht genau vorgesagt werden kann, nicht aber daß gar nicht gemessen werden könne.

Sry, lang ists her, aber eine Messung in der Tiefe beeinflusst doch das Ergebnis, oder wie war das?

wry
2007-09-21, 00:26:18
Danke erstmal.
Hab das Problem hier rein gepostet, da ich dachte im Programmierforum würden sich einige Informatiker rumtreiben, die damit vielleicht etwas anfangen können. Scheint ja einige zu geben.


Beispielaufgabe wäre folgende:

Ein Zustand |psi> in der Basis {|00>,|01>,|10>,|11>} ist gegeben:

|psi> = 1/2 * ( |00> + |01> + |10> + |11> )

Meine Aufgabe besteht darin den Zustand |psi> einer Bell-Messung zu unterziehen, also eine Messung in der Basis
{1/sqrt(2)*( |00> + |11>), =: |phi+>
1/sqrt(2)*( |00> - |11>), =: |phi->
1/sqrt(2)*( |01> + |10>), =: |psi+>
1/sqrt(2)*( |01> - |10>)}. =: |psi->

Also würde ich die Wahrscheinlichkeiten für die Messergebnisse wie folgt ermitteln (wenn ich den Gast richtig verstanden hab):


P_{Y,j} = <Y_j|psi> = sum_i=1^4 a_{X,i} <Y_j|X_i>



P_{phi+} = <phi+|psi> = [ 1/sqrt(2)*( |00> + |11>) ] * [ 1/2 * ( |00> + |01> + |10> + |11> ) ] = 1/(2*sqrt(2)) + 1/(2*sqrt(2) = 1/sqrt(2)
P_{phi-} = <phi-|psi> = ... = 0
P_{psi+} = <psi+|psi> = ... = 1/sqrt(2)
P_{psi-} = <psi-|psi> = ... = 0


Ist das korrekt? Kann es sein dass ich die erhaltenen Wahrscheinlichkeiten noch quadrieren muss, damit ich Summe = 1 erhalte?

Gast
2007-09-21, 00:41:56
Erlaube mir bitte nochmal die Frage: Wie misst du das? In der Praxis? :)

wry
2007-09-21, 00:46:29
Erlaube mir bitte nochmal die Frage: Wie misst du das? In der Praxis? :)

Tut mir leid, dass weiß ich selbst nicht.
Bin leider kein Physiker und muss die Aufgabe nur in der Theorie lösen. :smile:

Coda
2007-09-21, 01:22:39
Was ist das Problem beim Messen? Es wird dadurch der Superzustand zwar aufgelöst, aber anders bekommt man ja nie ein Ergebnis.

Das was man misst ist dann mit einer gewissen Wahrscheinlichkeit das richtige (und zwar ist diese deutlich >50%, sonst ergibt es ja keinen Sinn).

Gast
2007-09-21, 01:33:39
Was ist das Problem beim Messen? Es wird dadurch der Superzustand zwar aufgelöst, aber anders bekommt man ja nie ein Ergebnis.

quark (http://www.google.de/search?rls=de&q=messung+elektronen&ie=utf-8&oe=utf-8&lr=lang_de)

Das was man misst ist dann mit einer gewissen Wahrscheinlichkeit das richtige (und zwar ist diese deutlich >50%, sonst ergibt es ja keinen Sinn).

(Hilfs-) Wissenschaft Mathematik oder angewandte Physik? Im Threadtitel steht "Messung" - und das wird nix.

Coda
2007-09-21, 01:37:30
quark (http://www.google.de/search?rls=de&q=messung+elektronen&ie=utf-8&oe=utf-8&lr=lang_de)
Ich weiß wirklich nicht auf was du hinaus willst? Ein Quantencomputer muss die Quantenzustände doch gar nicht mit Elektronen repräsentieren.

Edit: Ich glaube ich weiß was du willst, aber das ist doch pedantisch. An dem messen braucht man sich jetzt wirklich nicht aufhalten, ich versteh auch so ganz gut was er will.

wry
2007-09-21, 20:24:48
Kann vielleicht jemand mein Ergebnis vom Post weiter oben bestätigen?

Ich hätte auch noch eine weitere Frage, wie muss ich vorgehen wenn ich zb. nur das erste oder nur das zweite qubit von |psi> in der Bell-Basis messen möchte? Steh da momentan auf der Leitung. ;(

Gast
2007-09-22, 00:16:48
Ist das korrekt? Kann es sein dass ich die erhaltenen Wahrscheinlichkeiten noch quadrieren muss, damit ich Summe = 1 erhalte?ja richtig, hatte ich vergessen:

P_{Y,j} = |<Y_j|psi>|^2 = |sum_i=1^4 a_{X,i} <Y_j|X_i>|^2

Gast
2007-09-22, 00:21:30
Sry, lang ists her, aber eine Messung in der Tiefe beeinflusst doch das Ergebnis, oder wie war das?der Meßvorgang ändert den Zustand des Systems, ja, falls du das gemeint haben solltest.

Führst du das durch, was wry als "Bell-Messung" bezeichnet, und erhältst als Meßergebnis z.B. den Zustand |phi+>, dann geht das System aus dem Ausgangszustand |psi> in ebendiesen Zustand |phi+> über. Man sagt auch, es wird durch die Messung in diesem präpariert.

wry
2007-09-22, 19:00:03
@Gast
Danke für deine Hilfe.

Vielleicht noch jemand eine Idee wie man nur 1 qubit eines 2-qubit Systems in einer anderen Basis messen könnte?

Gast
2007-09-22, 22:27:44
der Meßvorgang ändert den Zustand des Systems, ja, falls du das gemeint haben solltest.

Führst du das durch, was wry als "Bell-Messung" bezeichnet, und erhältst als Meßergebnis z.B. den Zustand |phi+>, dann geht das System aus dem Ausgangszustand |psi> in ebendiesen Zustand |phi+> über. Man sagt auch, es wird durch die Messung in diesem präpariert.

Trotzdem nur ne Ungleichung mit zwei Unbekannten (präpariert wird wohl eine Variable). Aber THX für die Erklärung. Physikalisch gemessen wurde hier allerdings nichts, der TS sprach von einem Beispiel, und ich hoffte schon, hier würde z.B. der Versuchsaufbau (mit Pix o.ä.) gepostet.

Gast
2007-09-22, 23:25:52
@Gast
Danke für deine Hilfe.

Vielleicht noch jemand eine Idee wie man nur 1 qubit eines 2-qubit Systems in einer anderen Basis messen könnte?meinst du damit eine 1-Qubit-Basis? Also der Form { |Y_i> = a_i |0> + b_i |1> }? Z.B.

{ |psi+> = 1/sqrt(2) |0> + 1/sqrt(2) |1>,
|psi-> = 1/sqrt(2) |0> - 1/sqrt(2) |1> }

? Dann ist es ganz ähnlich wie in der 2-Qubit-Basis: sei |psi> der aktuelle Zustand des einen Qubits, dann ist die Wahrscheinlichkeit, bei einer Messung |psi+> bzw. |psi-> vorzufinden,

P+/i = |<psi+/-|psi>|^2

Oder geht es dir um die Messung eines einzelnen Qubits in einer 2-Qubit-Basis? Das geht gar nicht: wenn die Basiszustände dieser Basis Superpositionen von { |00>, |01>, |10>, |11> } sind, dann ist es nicht mehr möglich, innerhalb eines Basiszustand den beiden Einzelqubits eindeutige Zustände zuzuschreiben, man hat dann einen verschränkten Zustand beider Qubits.

Gast
2007-09-22, 23:29:50
Trotzdem nur ne Ungleichung mit zwei Unbekannten (präpariert wird wohl eine Variable). Aber THX für die Erklärung. Physikalisch gemessen wurde hier allerdings nichts, der TS sprach von einem Beispiel, und ich hoffte schon, hier würde z.B. der Versuchsaufbau (mit Pix o.ä.) gepostet.du hast aber nicht nur angemerkt, daß es hier keine konkrete praktische Messung gehe, sondern hast auch angedeutet, daß es eine solche deiner Ansicht nach gar nicht geben könne ("das wird nix"). Wie gelangst du zu dieser Schlußfolgerung?

Gast
2007-09-23, 00:13:05
Willst du mich veralbern?

wry
2007-09-23, 17:54:47
@Gast

Ja genau, deinen ersten Fall meinte ich. Ich glaub ich habs jetzt begriffen, danke! :smile:


Um jetzt noch ein Beispiel zu machen, ich habe 2 qubits im Zustand |alpha>:

|alpha> = ( 0 0 1/sqrt(2) -1/sqrt(2) )

Ich will das erste qubit von |alpha> in der Basis {|0>,|1>} messen.

----

Ich hatte Probleme herauszufinden wie man zb das erste qubit aus dem 2-qubit Zustand herausliest um es dann messen zu können.
Aber ich glaube ich hab das jetzt begriffen, man addiert einfach die Wahrscheinlichkeiten richtig zusammen und erhält es somit.


z.B. qubit 1 aus |alpha> auslesen:
|alpha> = Summe von
0 * |00> -\
0 * |01> --> Wahrscheinlichkeit für erstes qubit im Zustand |0>: 0 + 0 = 0
1/sqrt(2) * |10> -\
-1/sqrt(2) * |11> --> Wahrscheinlichkeit für erstes qubit im Zustand |1>: |1/sqrt(2)|^2 + |-1/sqrt(2)|^2 = 1

Erstes qubit von |alpha> wäre also:
|alpha_1> := 0 * |0> + 1 * |1> = |1>


Jetzt kann ich |alpha_1> mit (Gasts Formel) messen und würde die Wahrscheinlichkeiten P_{|0>}=0, P_{|1>}=1 und |1> als Ergebniszustand erhalten.

Gast
2007-09-23, 21:07:42
@Gast

Ja genau, deinen ersten Fall meinte ich. Ich glaub ich habs jetzt begriffen, danke! :smile:


Um jetzt noch ein Beispiel zu machen, ich habe 2 qubits im Zustand |alpha>:

|alpha> = ( 0 0 1/sqrt(2) -1/sqrt(2) )

Ich will das erste qubit von |alpha> in der Basis {|0>,|1>} messen.
du meinst wohl

|alpha> = 1/sqrt(2) |10> - 1/sqrt(2) |11>

wenn du da jetzt Wahrscheinlichkeiten für die Einzel-Qubits berechnen willst, mußt du mit der Dichtematrix arbeiten. Die Dichtematrix des Gesamtsystems ist

rho_ges = |alpha><alpha|

= 1/2 |10><10| - 1/2 |10><11| - 1/2 |11><10| + 1/2 |11><11|

Um daraus jetzt die Dichtematrix rho_1 des ersten Qubits zu gewinnen, mußt du über alle möglichen Zustände des zweiten Qubits (also |0_2> und |1_2>, Zustände der Einzelqubits tragen im folgenden einen Index, damit man erkennt zu welchem Qubit sie gehören) summieren ("ausspuren"):

rho_1 = <0_2| rho_ges |0_2> + <1_2| rho_ges |1_2>

= 1/2 <0_2|10><10|0_2> - 1/2 <0_2|10><11|0_2> - 1/2 <0_2|11><10|0_2> + 1/2 <0_2|11><11|0_2>
+ 1/2 <1_2|10><10|1_2> - 1/2 <1_2|10><11|1_2> - 1/2 <1_2|11><10|1_2> + 1/2 <1_2|11><11|1_2>

Durch die Skalarproduktbildung der 2-Qubit-Zustände mit den 1-Qubit-Zuständen des zweiten Qubits werden alle Summanden mit Ausnahme des ersten und letzen Null:

rho_1 = 1/2 <0_2|10><10|0_2> + 1/2 <1_2|11><11|1_2>

Wertet man auch bei diesen die Skalarprodukte aus (es gilt: |10> = |1_1>|0_2>, |11> = |1_1>|1_2>, daraus folgt <10|0_2> = <1_1|, <11|1_2> = <1_1|), so ergibt sich:

rho_1 = 1/2 |1_1><1_1| + 1/2 |1_1><1_1| = |1><1|

wobei ich beim letzten Ergebnis den Qubit-Index weggelassen habe, weil klar ist um welches Qubit es sich handelt.

Um jetzt die Wahrscheinlichkeit für einen 1-Qubit-Zustand |psi> des ersten Qubits zu erhalten, muß man den Erwartungswert der Dichtematrix für diesen Zustand bilden:

P_psi = <psi|rho_1|psi>

Man erkennt leicht, daß die Wahrscheinlichkeit für |0>:

P_0 = <0|1><1|0> = 0

ist, und die für |1>:

P_1 = <1|1><1|1> = 1

ungefähr so wie du es selbst berechnet hast :)

wry
2007-09-23, 23:48:58
Hmm, das sieht jetzt schon ein bisschen komplizierter aus ..

Ich zerbrech mir gerade seit längerer Zeit den Kopf wie die Vektoren |0_2> und |1_2> aussehen und komm einfach nicht drauf. :redface:

Ich dachte zuerst:
|0_2> = (|00>+|10>)
|1_2> = (|01>+|11>)

Aber als ichs nachrechnen wollte, hab ich bemerkt, dass ich auf dem Hohlweg bin -_-

Gast
2007-09-24, 15:03:42
Hmm, das sieht jetzt schon ein bisschen komplizierter aus ..

Ich zerbrech mir gerade seit längerer Zeit den Kopf wie die Vektoren |0_2> und |1_2> aussehen und komm einfach nicht drauf. :redface:

Ich dachte zuerst:
|0_2> = (|00>+|10>)
|1_2> = (|01>+|11>)|0_2> und |1_2> sind Ein-Qubit-Zustände, |00>, |10>, |01>, |11> sind Zwei-Qubit-Zustände, deine Konstruktion kann von daher gar nicht funktionieren. Das _2 ist nur ein Index, der angibt zu welchem Qubit der Zustand gehört (das 2. Qubit). Wenn das feststeht, kannst du den Index auch weglassen und einfach |0> und |1> schreiben.
Alternativ könntest du auch schreiben |0_2> = |Qubit 2 ist im Zustand 0> und |1_2> = |Qubit 2 ist im Zustand 1>.
Die vier 2-Qubit-Zustände sind Tensorprodukte aus Ein-Qubit-Zuständen der beiden Einzel-Qubits:

|00> = |0_1>|0_2>

usw.

wry
2007-09-24, 19:50:45
Wie ist dann <10|0_2> definiert?

Der Vektor <10| hat ja dann 4 Komponenten und |0_2> nur 2? Laut der Definition vom inner product vor mir, sollten die beiden Vektoren gleich viele Einträge haben und das Ergebnis ist eine Zahl und kein Vektor wie bei dir. :confused:

Welche Unterlagen verwendest du eigentlich? Kannst du mir da vielleicht was empfehlen?


Edit:
Argh, jetzt hab ichs, du hasts ja schon dazugeschrieben. Bin echt ne Niete was das Mathe Zeug angeht. :mad:

<10|0_2> = <1|<0|0> = <1| * 1 = <1|

Danke nochmal für deine Mühe.

Gast
2007-09-25, 00:37:14
Wie ist dann <10|0_2> definiert?

Der Vektor <10| hat ja dann 4 Komponenten und |0_2> nur 2? Laut der Definition vom inner product vor mir, sollten die beiden Vektoren gleich viele Einträge haben und das Ergebnis ist eine Zahl und kein Vektor wie bei dir. :confused:Der Vektor |10> ist definiert auf dem Hilbertraum des 2-Qubit-Systems, und der ist das Tensorprodukt der Hilbert-Räume der beiden Einzel-Qubits. Dadurch ist dann auch das Skalarprodukt zwischen Vektoren im Gesamt-Hilbertraum H_ges und Vektoren in den beiden Teilräumen H1 und H2 definiert. Dergestalt, daß das Skalarprodukt aus einem Vektor aus H_ges und einem aus H1 einen Vektor in H2 ergibt, und umgekehrt.

Welche Unterlagen verwendest du eigentlich? Kannst du mir da vielleicht was empfehlen?Vorlesung Theoretische Physik III, Uni Duisburg Wintersemester 99/2000. :)

Sollte aber auch in Quantenmechanik-Lehrbüchern wie dem Schwabl, Messiah, Greiner oder Nolting zu finden sein.

Kannst auch mal hier:

http://theory.gsi.de/~vanhees/faq/qm/node35.html

gucken.

wry
2007-09-25, 19:59:52
Meine Mathekenntnisse werden wohl nicht ausreichen um Physikerliteratur zu verstehen, aber werds trotzdem mal versuchen. Thx :smile: